Integral problem about probability












0












$begingroup$


Let $h(x)=frac{f'(x)^2}{f(x)},$ and $f(x)$ is the probability density function of r.v. $X$.



If $int_{-infty}^{infty}(1+x^2)h(x)dx<infty$, do we have $int_{-infty}^{infty} xh(x)dx=0$?



Many thanks










share|cite|improve this question











$endgroup$








  • 1




    $begingroup$
    I am not sure whether one could say an indefinite integral is bounded or not, i.e. I suppose there are some borders of integration missing.
    $endgroup$
    – mrtaurho
    Dec 23 '18 at 12:49






  • 1




    $begingroup$
    The integration could be regarded as expectation. I should put the borders from $-infty$ to $infty$
    $endgroup$
    – FoxFlu
    Dec 23 '18 at 12:54










  • $begingroup$
    Note that $int_{-infty}^infty h(x),mathrm{d}xgt0$. Furthermore, if $f(x)$ is a probability density function, then $f(x-1)$ is also. If $int_{-infty}^infty x h(x),mathrm{d}x=0$, then $int_{-infty}^infty x h(x-1),mathrm{d}x=int_{-infty}^infty h(x),mathrm{d}xgt0$. Thus, it is not true for all probability densities.
    $endgroup$
    – robjohn
    Dec 23 '18 at 13:39










  • $begingroup$
    For exponential distribution with pdf $f(x)=e^{-x}$ ($x>0$) we get $h(x)=f(x)$ and $int_{-infty}^infty xh(x)dx=1$.
    $endgroup$
    – NCh
    Dec 24 '18 at 3:25
















0












$begingroup$


Let $h(x)=frac{f'(x)^2}{f(x)},$ and $f(x)$ is the probability density function of r.v. $X$.



If $int_{-infty}^{infty}(1+x^2)h(x)dx<infty$, do we have $int_{-infty}^{infty} xh(x)dx=0$?



Many thanks










share|cite|improve this question











$endgroup$








  • 1




    $begingroup$
    I am not sure whether one could say an indefinite integral is bounded or not, i.e. I suppose there are some borders of integration missing.
    $endgroup$
    – mrtaurho
    Dec 23 '18 at 12:49






  • 1




    $begingroup$
    The integration could be regarded as expectation. I should put the borders from $-infty$ to $infty$
    $endgroup$
    – FoxFlu
    Dec 23 '18 at 12:54










  • $begingroup$
    Note that $int_{-infty}^infty h(x),mathrm{d}xgt0$. Furthermore, if $f(x)$ is a probability density function, then $f(x-1)$ is also. If $int_{-infty}^infty x h(x),mathrm{d}x=0$, then $int_{-infty}^infty x h(x-1),mathrm{d}x=int_{-infty}^infty h(x),mathrm{d}xgt0$. Thus, it is not true for all probability densities.
    $endgroup$
    – robjohn
    Dec 23 '18 at 13:39










  • $begingroup$
    For exponential distribution with pdf $f(x)=e^{-x}$ ($x>0$) we get $h(x)=f(x)$ and $int_{-infty}^infty xh(x)dx=1$.
    $endgroup$
    – NCh
    Dec 24 '18 at 3:25














0












0








0





$begingroup$


Let $h(x)=frac{f'(x)^2}{f(x)},$ and $f(x)$ is the probability density function of r.v. $X$.



If $int_{-infty}^{infty}(1+x^2)h(x)dx<infty$, do we have $int_{-infty}^{infty} xh(x)dx=0$?



Many thanks










share|cite|improve this question











$endgroup$




Let $h(x)=frac{f'(x)^2}{f(x)},$ and $f(x)$ is the probability density function of r.v. $X$.



If $int_{-infty}^{infty}(1+x^2)h(x)dx<infty$, do we have $int_{-infty}^{infty} xh(x)dx=0$?



Many thanks







real-analysis calculus probability integration probability-theory






share|cite|improve this question















share|cite|improve this question













share|cite|improve this question




share|cite|improve this question








edited Dec 23 '18 at 12:55







FoxFlu

















asked Dec 23 '18 at 12:47









FoxFluFoxFlu

12




12








  • 1




    $begingroup$
    I am not sure whether one could say an indefinite integral is bounded or not, i.e. I suppose there are some borders of integration missing.
    $endgroup$
    – mrtaurho
    Dec 23 '18 at 12:49






  • 1




    $begingroup$
    The integration could be regarded as expectation. I should put the borders from $-infty$ to $infty$
    $endgroup$
    – FoxFlu
    Dec 23 '18 at 12:54










  • $begingroup$
    Note that $int_{-infty}^infty h(x),mathrm{d}xgt0$. Furthermore, if $f(x)$ is a probability density function, then $f(x-1)$ is also. If $int_{-infty}^infty x h(x),mathrm{d}x=0$, then $int_{-infty}^infty x h(x-1),mathrm{d}x=int_{-infty}^infty h(x),mathrm{d}xgt0$. Thus, it is not true for all probability densities.
    $endgroup$
    – robjohn
    Dec 23 '18 at 13:39










  • $begingroup$
    For exponential distribution with pdf $f(x)=e^{-x}$ ($x>0$) we get $h(x)=f(x)$ and $int_{-infty}^infty xh(x)dx=1$.
    $endgroup$
    – NCh
    Dec 24 '18 at 3:25














  • 1




    $begingroup$
    I am not sure whether one could say an indefinite integral is bounded or not, i.e. I suppose there are some borders of integration missing.
    $endgroup$
    – mrtaurho
    Dec 23 '18 at 12:49






  • 1




    $begingroup$
    The integration could be regarded as expectation. I should put the borders from $-infty$ to $infty$
    $endgroup$
    – FoxFlu
    Dec 23 '18 at 12:54










  • $begingroup$
    Note that $int_{-infty}^infty h(x),mathrm{d}xgt0$. Furthermore, if $f(x)$ is a probability density function, then $f(x-1)$ is also. If $int_{-infty}^infty x h(x),mathrm{d}x=0$, then $int_{-infty}^infty x h(x-1),mathrm{d}x=int_{-infty}^infty h(x),mathrm{d}xgt0$. Thus, it is not true for all probability densities.
    $endgroup$
    – robjohn
    Dec 23 '18 at 13:39










  • $begingroup$
    For exponential distribution with pdf $f(x)=e^{-x}$ ($x>0$) we get $h(x)=f(x)$ and $int_{-infty}^infty xh(x)dx=1$.
    $endgroup$
    – NCh
    Dec 24 '18 at 3:25








1




1




$begingroup$
I am not sure whether one could say an indefinite integral is bounded or not, i.e. I suppose there are some borders of integration missing.
$endgroup$
– mrtaurho
Dec 23 '18 at 12:49




$begingroup$
I am not sure whether one could say an indefinite integral is bounded or not, i.e. I suppose there are some borders of integration missing.
$endgroup$
– mrtaurho
Dec 23 '18 at 12:49




1




1




$begingroup$
The integration could be regarded as expectation. I should put the borders from $-infty$ to $infty$
$endgroup$
– FoxFlu
Dec 23 '18 at 12:54




$begingroup$
The integration could be regarded as expectation. I should put the borders from $-infty$ to $infty$
$endgroup$
– FoxFlu
Dec 23 '18 at 12:54












$begingroup$
Note that $int_{-infty}^infty h(x),mathrm{d}xgt0$. Furthermore, if $f(x)$ is a probability density function, then $f(x-1)$ is also. If $int_{-infty}^infty x h(x),mathrm{d}x=0$, then $int_{-infty}^infty x h(x-1),mathrm{d}x=int_{-infty}^infty h(x),mathrm{d}xgt0$. Thus, it is not true for all probability densities.
$endgroup$
– robjohn
Dec 23 '18 at 13:39




$begingroup$
Note that $int_{-infty}^infty h(x),mathrm{d}xgt0$. Furthermore, if $f(x)$ is a probability density function, then $f(x-1)$ is also. If $int_{-infty}^infty x h(x),mathrm{d}x=0$, then $int_{-infty}^infty x h(x-1),mathrm{d}x=int_{-infty}^infty h(x),mathrm{d}xgt0$. Thus, it is not true for all probability densities.
$endgroup$
– robjohn
Dec 23 '18 at 13:39












$begingroup$
For exponential distribution with pdf $f(x)=e^{-x}$ ($x>0$) we get $h(x)=f(x)$ and $int_{-infty}^infty xh(x)dx=1$.
$endgroup$
– NCh
Dec 24 '18 at 3:25




$begingroup$
For exponential distribution with pdf $f(x)=e^{-x}$ ($x>0$) we get $h(x)=f(x)$ and $int_{-infty}^infty xh(x)dx=1$.
$endgroup$
– NCh
Dec 24 '18 at 3:25










0






active

oldest

votes












Your Answer








StackExchange.ready(function() {
var channelOptions = {
tags: "".split(" "),
id: "69"
};
initTagRenderer("".split(" "), "".split(" "), channelOptions);

StackExchange.using("externalEditor", function() {
// Have to fire editor after snippets, if snippets enabled
if (StackExchange.settings.snippets.snippetsEnabled) {
StackExchange.using("snippets", function() {
createEditor();
});
}
else {
createEditor();
}
});

function createEditor() {
StackExchange.prepareEditor({
heartbeatType: 'answer',
autoActivateHeartbeat: false,
convertImagesToLinks: true,
noModals: true,
showLowRepImageUploadWarning: true,
reputationToPostImages: 10,
bindNavPrevention: true,
postfix: "",
imageUploader: {
brandingHtml: "Powered by u003ca class="icon-imgur-white" href="https://imgur.com/"u003eu003c/au003e",
contentPolicyHtml: "User contributions licensed under u003ca href="https://creativecommons.org/licenses/by-sa/3.0/"u003ecc by-sa 3.0 with attribution requiredu003c/au003e u003ca href="https://stackoverflow.com/legal/content-policy"u003e(content policy)u003c/au003e",
allowUrls: true
},
noCode: true, onDemand: true,
discardSelector: ".discard-answer"
,immediatelyShowMarkdownHelp:true
});


}
});














draft saved

draft discarded


















StackExchange.ready(
function () {
StackExchange.openid.initPostLogin('.new-post-login', 'https%3a%2f%2fmath.stackexchange.com%2fquestions%2f3050320%2fintegral-problem-about-probability%23new-answer', 'question_page');
}
);

Post as a guest















Required, but never shown

























0






active

oldest

votes








0






active

oldest

votes









active

oldest

votes






active

oldest

votes
















draft saved

draft discarded




















































Thanks for contributing an answer to Mathematics Stack Exchange!


  • Please be sure to answer the question. Provide details and share your research!

But avoid



  • Asking for help, clarification, or responding to other answers.

  • Making statements based on opinion; back them up with references or personal experience.


Use MathJax to format equations. MathJax reference.


To learn more, see our tips on writing great answers.




draft saved


draft discarded














StackExchange.ready(
function () {
StackExchange.openid.initPostLogin('.new-post-login', 'https%3a%2f%2fmath.stackexchange.com%2fquestions%2f3050320%2fintegral-problem-about-probability%23new-answer', 'question_page');
}
);

Post as a guest















Required, but never shown





















































Required, but never shown














Required, but never shown












Required, but never shown







Required, but never shown

































Required, but never shown














Required, but never shown












Required, but never shown







Required, but never shown







Popular posts from this blog

Plaza Victoria

In PowerPoint, is there a keyboard shortcut for bulleted / numbered list?

How to put 3 figures in Latex with 2 figures side by side and 1 below these side by side images but in...